Page 1 of 1

#22 - Global, Could Be True, List

Posted: Fri Aug 03, 2018 3:38 pm
by Dave Killoran
Complete Question Explanation
(The complete setup for this game can be found here: lsat/viewtopic.php?t=9142)

The correct answer choice is (D)

Again, most of the incorrect answer choices are eliminated by the initial inferences.

Answer choice (A) is incorrect because F and M cannot be prescribed together.

Answer choice (B) is incorrect because F and W cannot be prescribed together.

Answer choice (C) is incorrect because V and N cannot be prescribed together.

Answer choice (D) is the correct answer choice.

Answer choice (E) is incorrect because V and N cannot be prescribed together.